Un jeu

Bonjour,

Soit $m$ un entier naturel strictement supérieur à $1$.

Quelle condition $m$ doit-il remplir pour que la congruence $x^2\equiv -1\pmod{m}$ ait au moins une solution ?

(On pourrait me suspecter de présenter sous la forme d'un jeu une vraie question à laquelle je suis censée trouver moi-même la solution, pour obtenir facilement la réponse. Afin de prouver ma bonne foi, je vous donne ma solution, mais sous forme cryptée sinon ce jeu n'aurait aucun sens : $111$, solution dont j'expliquerai la signification plus tard.)

Merci d'avance à qui voudra bien jouer.
(En même temps, c'est peut-être très connu.)

Réponses

  • Hello Sneg,

    Je répond sous forme de code aussi : $2,0,1; 1,*;3,0$ :-D

    Oups j'ai mal lu le jeu :-D
  • Bonjour,

    Soit $b$ un entier.

    Pour $x=b^2+b+1$ on a $x^2=-1+(b^2+1)(b^2+2b +2)=-1 \mod(b^2+1).$
    Il suffit de choisir $m=b^2+1.$
  • Bonsoir Goleon et YvesM.

    YvesM, ta solution semble ne pas être générale.
    Par exemple, dans le cas où $m=13$, on a bien $5^2\equiv -1\pmod{13}$. Et $m$ ne vaut pas $b^2+1$.
  • Pour $p$ premier impair $z^2\equiv -1\bmod p$ a une solution ssi $p\equiv 1\bmod 4$, dans ce cas $(z^{p^{k-1}})^2\equiv -1\bmod p^k$

    On en déduit que $x^2\equiv -1\bmod m$ a une solution ssi $m = 2^r \prod_j p_j^{k_j}$ où $p_j\equiv 1\bmod 4$ et $r\in 0,1$
  • Particulièrement intéressant, reuns.
  • Sneg,
    Comme c'est un jeu, je me permets de faire mumuse ci-dessous. Je pense que tu me pardonneras le changement de notations que je vais faire : je vais noter $a$ au lieu de $m$ et $b$ une racine carrée de $-1$ modulo $a$ : $b^2 \equiv - 1 \bmod a$.
    Et je vais essayer de t'illustrer, SANS EXPLICATION, pourquoi $a$ est une somme PRIMITIVE de deux carrrés i.e. $a$ s'écrit $a = x^2 + y^2$ avec $\gcd(x,y) = 1$. Et on a une réciproque (facile) : si $a$ est une somme primitive de 2 carrés, alors $-1$ est un carré modulo $a$.

    Je prends $a$ conformément au post de reuns et je fais déterminer une racine carrée de $-1$ modulo $a$.
    [color=#000000]> a := 2 * 5^2 * 13^3 * 17 ;   
    > a ;
    1867450     
    > b := Modsqrt(-1, a) ;
    > b ;
    1001593
    [/color]
    
    L'algorithme que je vais faire tourner nécessite que $b$ soit strictement plus petit que la moitié de $a$. Je corrige $b$ en conséquence.
    [color=#000000]> b := a - b ;
    > b ;
    865857
    > (b^2 + 1) mod a ;
    0
    [/color]
    
    Je fais tourner l'algorithme dEuclide du calcul de pgcd de $a, b$. Ce pgcd est 1 (facile) mais ce qui m'intéresse, c'est la suite des restes $r_0, r_1, \cdots, r_n, r_{n+1} = 0$. Et un peu la suite des quotients $q_1, \cdots, q_n$.

    Je stocke ce petit monde dans $R = (r_1, \cdots, r_n)$ et $Q = (q_1, \cdots, q_n)$ (je ne mets pas $r_0$ dans $R$).
    [color=#000000]> r0 := a ;  r1 := b ;
    > R := [] ;  Q := [] ;
    > while r1 ne 0 do
    while> Append(~R, r1) ;
    while> q, r := Quotrem(r0,r1) ;
    while> Append(~Q, q) ;
    while> r0 := r1 ; r1 := r ;
    while> end while ;
    [/color]
    
    Premier miracle : la suite des quotients est une suite mirroir. Et la longueur $n$ est PAIRE.
    [color=#000000]> Q ;                                
    [ 2, 6, 2, 1, 1, 1, 3, 3, 3, 3, 1, 1, 1, 2, 6, 2 ]
    > n := #Q ;
    > n ;
    16
    [/color]
    
    Mais le véritable miracle est ci-dessous : c'est que $a$ est la somme (primitive) des carrés $a = r_m^2 + r_{m+1}^2$ où $m = n/2$.
    [color=#000000]> R ;
    [ 865857, 135736, 51441, 32854, 18587, 14267, 4320, 1307, 399, 110, 69, 41, 28, 13, 2, 1 ]
    > m := ExactQuotient(n,2) ;
    > a ;
    1867450
    > R[m]^2 + R[m+1]^2 ;
    1867450
    > Gcd(R[m], R[m+1]) ;
    1
    [/color]
    
    Play again avec un $a$ plus grand. Ici pas besoin de corriger $b$.
    [color=#000000]> P := [p : p in PrimesInInterval(2,100) | p mod 4 eq 1] ;> P ;
    [ 5, 13, 17, 29, 37, 41, 53, 61, 73, 89, 97 ]
    > 
    > P := [p : p in PrimesInInterval(5,100) | p mod 4 eq 1] ;
    > P ;                                                     
    [ 5, 13, 17, 29, 37, 41, 53, 61, 73, 89, 97 ]
    > 
    > a := 5^3 * 13^3 * 41^3 * 61 * 73^2 * 97 ;               
    > a ;
    596813900214605125
    > b := Modsqrt(-1, a) ;                                   
    > b ;
    94051846532879568
    > a - b ;     
    502762053681725557
    > (b^2 + 1) mod a ;   
    0
    > r0 := a ;  r1 := b ;
    > R := [] ;  Q := [] ;
    > while r1 ne 0 do
    while> Append(~R, r1) ;
    while> q, r := Quotrem(r0,r1) ;
    while> Append(~Q, q) ;
    while> r0 := r1 ; r1 := r ;
    while> end while ;
    > Q ;
    [ 6, 2, 1, 8, 2, 2, 12, 1, 1, 1, 1, 2, 1, 2, 107, 3, 1, 1, 1, 1, 1, 1, 3, 107, 2, 1, 2, 1, 1, 1, 1, 12, 2, 2, 8, 1, 2, 6 ]
    > n := #Q ;
    > n ;
    38
    > R ;
    [ 94051846532879568, 32502821017327717, 29046204498224134, 3456616519103583, 1393272345395470, 670071828312643, 53128688770184, 
    32527563070435, 20601125699749, 11926437370686, 8674688329063, 3251749041623, 2171190245817, 1080558795806, 10072654205, 
    2784795871, 1718266592, 1066529279, 651737313, 414791966, 236945347, 177846619, 59098728, 550435, 202183, 146069, 56114, 33841, 
    22273, 11568, 10705, 863, 349, 165, 19, 13, 6, 1 ]
    > m := ExactQuotient(n,2) ;
    > R[m] ;             
    651737313
    > R[m+1] ; 
    414791966
    > a eq R[m]^2 + R[m+1]^2 ;
    true
    > Gcd(R[m], R[m+1]) ;
    1
    [/color]
    
    Si tu veux en savoir plus, fais moi signe. J'ai un exercice corrigé pour des petits (que je tire d'un article de Wagon ``The Euclidean Algorithm Strikes again'', la référence exacte est dans l'exercice).
  • Bonsoir, claude quitté

    J'apprécie toujours que des intervenants répondent à mes questions.
    Cela dit, quand c'est toi qui le fais et que je vois ton nom apparaître, je me dis qu'il va se passer quelque chose. Et c'est encore une fois le cas aujourd'hui. (D'une certaine façon, cela me fait également peur, car je sais que l'écart qui sépare mes connaissances mathématiques des tiennes n'a pas de mesure.)

    "Si $a$ est une somme primitive de deux carrés, alors $-1$ est un carré modulo $a$."

    Il se fait que j'avais remarqué cette propriété, mais sans pouvoir la démontrer.
    Alors, j'ai cherché un peu partout une démonstration. Et tout ce que j'ai trouvé, je l'ai trouvé chez Gauss - doit-on s'en étonner ? - qui écrit ceci au n° $111$ de ses Disquisitiones Arithmeticae :
    "Au reste on déduit facilement de ce qui précède cette règle générale : $-1$ est résidu (quadratique) de tous les nombres qui ne sont divisibles ni par $4$, ni par aucun nombre de la forme $4n+3$. Il est non-résidu de tous les autres."

    En posant ma question sous la forme d'un jeu, j'espérais secrètement que quelqu'un parte dans la direction de la propriété que j'avais remarquée. Et tu l'as fait ! Cela doit être le miracle de Noël.

    Cela me ferait très plaisir si tu pouvais m'orienter vers une démonstration.
    Mille mercis.
  • Si $a=x^2+y^2$ avec $x$ et $y$ premiers entre eux, alors on a $x^2 + y^2 \equiv 0$ mod $a$, autrement dit $x^2 \equiv - y^2$ mod $a$. On a forcément ($x$ premier avec $a$) ou ($y$ premier avec $a$), sinon on déduit facilement de $a=x^2+y^2$ que $x$ et $y$ ne sont pas premiers entre eux. On peut supposer $y$ premier avec $a$, et donc invrsible modulo $a$, et on obtient $(x/y)^2 \equiv -1$ mod $a$.
  • $\def\F{\mathbb F}$Coucou Sneg,
    Je réponds après Poirot sans te donner vraiment la solution. Une précision : pas de négation ni de raisonnement par l'absurde avec mézigue. Pourquoi ? Parce que je n'aime pas et que je suis capricieux.

    $\bullet$ A. Si tu connais les nombres complexes. En présence de $x^2 + y^2 = 0$ dans $\C$ et de $x \ne 0$, que peux tu en déduire concernant $(x^{-1}y)^2$ ? Et comme je n'aime pas $x \ne 0$, je le remplace par la vraie vérité : $x$ est inversible.

    Dans l'histoire $a = x^2 + y^2$ avec $\gcd(x,y) = 1$, en passant dans $\Z/a\Z$, on obtient $x^2 + y^2 = 0$. On va pouvoir appliquer le trick ci-dessus. A condition de prouver que $x, y$ sont inversibles modulo $a$. Note : ils le sont tous les deux.
    Questions : pourrais tu montrer que $\gcd(a,x) = 1$ ET que $\gcd(a,y) = 1$ ? De la manière la plus directe. Et en déduire ce qu'il faut.

    $\bullet$ B. Du coup, je vais continuer à jouer. D'abord, j'ai relu l'article de Stan Wagon, The Euclidean Algorithm Strikes Again, Amer. Math. Monthly vol. 97, numéro 2, 1990. J'y ai vu, lorsque $a$ est un premier $p \equiv 1 \bmod 4$, disons que $p = 4k + 1$, que Gauss disposait d'une formule pour écrire $p$ sous la forme $x^2 + y^2$.
    $$
    x = {1 \over 2} {2k \choose k} \bmod p, \qquad y = (2k! \times x) \bmod p
    $$A CONDITION de prendre pour $\bmod$ l'opérateur de reste $r$ dans la division par $p$ vérifiant $|r| < p/2$. I.e. remplacer le reste habituel $r$ par $p-r$ si besoin. Cette formule de détermination de $(x,y)$ n'est pas efficace pour le calcul mais ce n'est pas son but.
    [color=#000000]> k := 15 ;                                                             
    > p := 4*k + 1 ;
    > p ;
    61
    > Reste := func < m |  2*r lt p select r else p-r where r is m mod p > ;
    > x := Reste(ExactQuotient(Binomial(2*k, k), 2)) ;
    > y := Reste(Factorial(2*k) * x) ;
    > x ;
    5
    > y ;
    6
    > p ;
    61
    > x^2 + y^2 ;
    61
    [/color]
    

    $\bullet$ C. J'ai lu aussi le coup suivant, toujours dans le contexte $p \equiv 1 \bmod 4$. Soit $N_p$ le nombre de points de $y^2 = x^3 -x$ sur $\F_p = \Z/p\Z$. Alors :
    $$
    x = \left| {N_p - p \over 2} \right| \quad \text{ intervient dans} \quad p = x^2 + y^2
    $$
    [color=#000000]> EllipticCurve([-1, 0]) ;         
    Elliptic Curve defined by y^2 = x^3 - x over Rational Field
    > p := 61 ;
    > Fp := GF(p) ;
    > Ep := EllipticCurve([Fp| -1, 0]) ;
    > Ep ;
    Elliptic Curve defined by y^2 = x^3 + 60*x over GF(61)
    > Np := #Ep - 1 ;
    > Np ;
    71
    > (Np - p) / 2 ;
    5
    [/color]
    
    $\bullet$ D. Pour épater ses ami(e)s. Les polynômes d'Euler. Tu connais ? Regarde bien la formation de
    $$
    f_4(q_1,q_2,q_3,q_4), \qquad \qquad f_5(q_1, q_2, q_3, q_4, q_5)
    $$
    [color=#000000]> f4 := func < q1,q2,q3,q4 | q1*q2*q3*q4 + q3*q4 + q1*q4 + q1*q2 + 1 > ;
    > f5 := func < q1,q2,q3,q4,q5 | q1*q2*q3*q4*q5 + q3*q4*q5 + q1*q4*q5 + q1*q2*q5 + q1*q2*q3 + q5 + q3 + q1 > ;
    [/color]
    
    Par exemple, pour $f_5$, on met le produit $q_1q_2q_3q_4q_5$ et ensuite on supprime deux $q_i$ consécutifs pour obtenir $q_3q_4q_5 + q_1q_4q_5 + \cdots $. Et on recommence (suppression de deux consécutifs) pour obtenir $q_5 + q_3 + q_1$.

    Je choisis maintenant 5 entiers $q_i \ge 1$ qui vont être les quotients dans l'algorithme d'Euclide de deux entiers définis plus loin. Attention : le dernier quotient doit être $\ge 2$. Dans une division euclidienne de longueur $\ge 2$, le dernier quotient n'est jamais 1.
    [color=#000000]> q1 := 3 ;  q2 := 7 ;  q3 := 1 ; q4 := 5 ; q5 := 2 ;
    [/color]
    
    Les deux entiers $a,b$ avec lesquels on va réaliser l'algorithme d'Euclide :
    [color=#000000]> a := f5(q1,q2,q3,q4,q5) ;
    > a ;
    319
    > b := f4(q2,q3,q4,q5) ;
    > b ;
    102
    [/color]
    
    L'algorithme d'Euclide :
    [color=#000000]> r0 := a ; r1 := b ;
    > Q := [] ;
    > while r1 ne 0 do
    while> q, r := Quotrem(r0,r1) ;
    while> Append(~Q, q) ;
    while> r0 := r1 ; r1 := r ;
    while> end while ;
    > Q ;
    [ 3, 7, 1, 5, 2 ]
    [/color]
    
    On retrouve bien nos $q_i$ de départ. Merci Euclide, Euler, Gauss et les autres.
  • Malheureusement, claude quitté, je ne suis déjà pas en mesure de répondre à ces premières questions.

    Aussi, je crois qu’il est préférable que j’en reste là. Pardonne-moi.

    Un tout grand merci à toi ; ainsi qu’à Poirot que je n’ai pas encore salué.
  • Sneg,
    Vraiment ? Peut-être que je n'aurais pas utiliser dû utiliser l'adjectif ``inversible'' ? Ci-dessous un petit quelque chose.

    (1) Supposons $x^2 + y^2 = 0$ dans $\C$ avec $x \ne 0$. On divise par $x^2$ ce qui donne $1 + (x^{-1}y)^2 = 0$ d'où une égalité de type convoité $z^2 = -1$.

    On peut zapper sur ce point (1) : c'était juste pour donner une idée.

    (2) Soit $a = x^2 + y^2$ avec $\gcd(x,y) = 1$. Montrons que $x,a$ sont premiers entre eux. Soit $d$ un diviseur commun à $x$ et $a$. A fortiori $d \mid x^2$ donc $d \mid a - x^2 = y^2$. Mais $x,y$ étant premiers entre eux, il en est de même de $x,y^2$. Puisque $d$ divise $x$ et $y^2$, on a $d = \pm 1$, ce qu'il fallait montrer.

    (3) Faisons comme dans le point (1) ou simulons le au sens du calcul modulo $a$. Comme $x,a$ sont premiers entre eux, il existe un entier $x'$ tel que $x' x \equiv 1 \bmod a$ (on dit que $x'$ est un inverse de $x$ modulo $a$). On a :
    $$
    x^2 + y^2 \equiv 0 \bmod a, \qquad x'x \equiv 1 \bmod a \qquad \text{d'où par multiplication par } x'^2 \qquad
    1 + (x'y)^2 \equiv 0 \bmod a
    $$On vient de trouver une racine carrée de $-1$ modulo $a$ : c'est $x'y$.

    Pareil, pas pareil ?
  • C’est très gentil à toi, claude quitté, de t'intéresser à mon petit problème. Cela me fait énormément plaisir.

    Pour ma part, il se fait que je suis en pleine préparation des examens de janvier, ce qui ne me laisse pas la moindre opportunité de me consacrer à des problèmes de mathématiques, y compris les miens propres. C’est pourquoi j’aurais souhaité que tu m’orientes simplement vers une démonstration.

    J’espère que tu peux me comprendre.

    Encore une fois, grand merci à toi.
    Joyeux Noël et bonne année 2020 !

    Sneg.
  • D'une façon générale et pour $m$ un nombre impaire le nombre de solutions de l'équation $x^2\equiv a\pmod m$ est : $$
    \prod_{p|m}\left(1+\left({\tfrac {a}{p}}\right)\right),
    $$ avec $\left({\tfrac {a}{p}}\right)$ le symbole de Legendre : $$
    \left(\frac ap\right)=\left\{\begin{array}{cl}1,&\mbox{si }
    p\nmid a\mbox{ et } a \mbox{ résidu quadratique (mod }p);\\ -1,
    &\mbox{si }p\nmid a \mbox{ et }a \mbox{ n'est pas un résidu quadratique (mod }
    p);\\ 0,&\mbox{si }p\mid a.\end{array}\right.
    $$ Donc pour que $x^2\equiv -1\pmod{m}$ ait une solution, il doit que $p\mid m \implies \left({\tfrac {a}{p}}\right) \neq -1$.
    Dans $\mathbb{Z}/m\mathbb{Z}$ on a : $-1=m-1$, puisque $p|m$, on a $p \nmid m-1$. le seul cas qui reste pour que $\left({\tfrac {a}{p}}\right) \neq -1$ c'est pour $m-1$ un résidu quadratique $\pmod p$

    Conclusion: Pour que $x^2\equiv -1\pmod{m}$ ait une solution il faut que $p|m \implies (\exists k \in \mathbb{N}), m-1=k^2 \pmod p$ et dans ce cas le nombre de solutions est $2^{\omega (m)}$, avec $\omega(m)$ est le nombre des diviseurs premiers distincts de $m$
  • @Lagrida
    Dans ce que tu as écrit il faut préciser que $p$ est un diviseur premier impair.

    De même $\omega(m)$ est le nombre des diviseurs premiers impairs distincts de $m$.
  • @jandri, j'ai oublié de mentionner que $m$ est impaire.
  • claude quitté a écrit:
    http://www.les-mathematiques.net/phorum/read.php?5,1907720,1908138#msg-1908138
    Si tu veux en savoir plus, fais moi signe. J'ai un exercice corrigé pour des petits (que je tire d'un article de Wagon ``The Euclidean Algorithm Strikes again'', la référence exacte est dans l'exercice)
    Bonjour Claude
    Moi je suis preneur de cet exercice en tant que petit !
    J'en profite pour souhaiter à Claude, et à tous les intervenants du forum, une excellente année 2020.
    Amicalement,
    Jean-éric.
  • Jean-éric,
    Bonne année 2020.
    J'attache l'exercice corrigé en question (laisse béton l'incohérence des dates due à des recompilations). L'article dont je me suis inspiré est en https://www.jstor.org/stable/2323912?seq=1 mais l'accès n'est pas gratuit.
  • Un grand Merci Claude.

    Pour l'article on peut s'inscrire sur Jstor sans payer et avoir accès gracieusement à la lecture de plusieurs articles payant par mois (6 je crois). Donc j'ai un accès limité mais gratuit à certains articles B-)

    Jean-éric.
Connectez-vous ou Inscrivez-vous pour répondre.